Combinatoires et probabilités

Exercices dans ce dossier

Exercice

Combinatoire : les nombres de Bell *

9 novembre 2022 12:22 — Par Patrice Lassère

Pour tout \(n\in\mathbb N^\star\), on désigne par \(B_n\) le nombre de partitions de l’ensemble \(\mathbb [1,\dots,n]\) avec par convention \(B_0=1\).

  1. Montrer que pour tout \(n\in\mathbb N\) : \(\displaystyle B_{n+1}=\sum_{k=0}^n C_n^k B_k.\)

  2. Montrer que le rayon de convergence \(R\) de la série génératrice exponentielle\(f(z)=\sum_{k=0}^\infty \frac{B_n}{n!}z^n\) de la suite \((B_n)_0^\infty\) est strictement positif et calculer \(f(z)\) pour \(\vert z\vert<R\).

  3. Montrer que\(\displaystyle B_n=\dfrac{1}{e}\sum_{k=0}^\infty \dfrac{k^n}{k!}.\)

Exercice

Autour du nombre de dérangements *

9 novembre 2022 12:22 — Par Patrice Lassère

On désigne par \(D_n\) (\(D_0=1\) par convention) le nombre de permutations de \(\mathscr S_n=\{1,2,\dots,n\}\) n’ayant pas de points fixes (i.e. le nombre de dérangements)

  1. Approche classique : Montrer que \[n!=\sum_{k=0}^nC_n^kD_{k}=\sum_{k=0}^nC_n^kD_{n-k}.{\text{($\star$)}}\] Calculer \(\displaystyle \sum_{k=0}^p (-1)^kC_n^kC_{n-k}^{p-k}\) pour \(0\leq p\leq n\) et en déduire que \[D_n=n!\left( \sum_{k=0}^n\dfrac{(-1)^k}{k!}\right). {\text{($\star$)}}\]

  2. Avec les séries entières : Calculer \(\displaystyle \sum_{k=0}^n C_n^kD_k\). Minorer la rayon de convergence de la série génératrice de \((D_n)_n\) : \(\displaystyle D(z)=\sum_{k\geq 0}\dfrac{D_kz^k}{k!}\) et donner une expression de \(D(z)\). Retrouver la valeur de \(D_n\) et montrer que \(D_k\) est la partie entière de \(\displaystyle \dfrac{k!}{e}+\dfrac{1}{2}.\)

  3. Troisième approche : Montrer que pour tout \(n\geq 2\) : \(D_{n+1}=n(D_n+D_{n-1})\), en déduire que \(D_n=nD_{n-1}+(-1)^n\) et retrouver la valeur de \(D_n\).

Exercice

Probabilité que deux entiers soient premiers entre-eux *

9 novembre 2022 12:23 — Par Patrice Lassère

On se propose de démontrer que la probabilité \(r_n\) que deux entiers pris au hasard dans \(\{1,\dots,n\}\) soient premiers entre-eux vérifie \[r_n=\dfrac{1}{n^2}\sum_{d\geq 1}\mu(d)E\left( \dfrac{n}{d}\right)^2\quad\text{et}\quad \lim_n r_n=\dfrac{6}{\pi^2}\] où l’application (c’est la fonction de möebius) \(\mu\ :\ \mathbb N^\star\to\ \{-1,0,1\}\) est définie par : \[\mu(n)=\begin{cases} &1\qquad\quad\text{ si }n=1,\\ &0\qquad\quad\text{ si }n\text{ possède au moins un facteur carré, }\\ &(-1)^k\quad\text{ si }n=p_1 \dots p_k\text{où les }p_i\text{ sont des nombres premiers distincts.} \end{cases}\] Soient \(p_1,\dots,p_k\) les nombres premiers \(\leq n\) et pour \(1\leq i\leq k\) : \[V_i:=\left\lbrace (a,b)\in\{1,\dots,n\}^2\ :\ p_i\text{ divise }a\text{ et }b\right\rbrace .\]

Montrer que

\[\begin{aligned}\text{card}\left( \bigcup_{i=1}^k V_i\right) &=\sum_{\emptyset\neq I\subset\{1,\dots,n\}}(-1)^{1+\text{card}(I)}\text{card}\left( \bigcap_{i\in I}V_i\right) \\ &=-\sum_{\emptyset\neq I\subset\{1,\dots,n\}} (-1)^{\text{card}(I)}E\left(\dfrac{n}{\prod_{i\in I}p_i}\right)^2\\ &=-\sum_{d=2}^n\mu(d)E\left( \dfrac{n}{d}\right)^2\end{aligned}\] Et en déduire \(r_n\).

Montrer que \(\displaystyle\left\vert r_n-\sum_{d=1}^n \dfrac{\mu(d)}{d^2}\right\vert=0\left( \dfrac{\log(n)}{n}\right).\)

Montrer que \(\displaystyle\sum_{n\geq 1}\dfrac{1}{n^2}\sum_{d\geq 1} \dfrac{\mu(d)}{d^2}=\sum_{i\geq 1}\sum_{l\text{ divise } i}\dfrac{\mu(l)}{i^2}=1.\)

Conclure.

Exercice

Dénombrement dans les groupes *

9 novembre 2022 12:23 — Par Patrice Lassère

Soient \(G\) un groupe fini, \(A,B\) deux parties de \(G\) telles que \[\text{card}(A)+\text{card}(B)>\text{card}(G).\] Montrer que \(AB=G\) (où \(AB=\{\ ab,\ a\in A,\ b\in B\}\)).

;
Success message!